banner



How To Find A Removable Discontinuity

You are using an out of date browser. It may non display this or other websites correctly.
Yous should upgrade or apply an alternative browser.
  • Forums
  • Homework Help
  • Calculus and Beyond Homework Aid

How do I decide where a function has a removable and a jump discontinuity?

  • Thread starter rowkem
  • Start appointment

Homework Statement

I am given the following function, piecewise:

f(x) = (-x+b) (10<1)
3 if x=ane
(-12/(10-b))-i (x>1, ten=/b)

I am asked:

ane) For what value(south) of 'b' does 'f' have a removable discontinuity at 1?
ii) For what value(s) of 'b' does 'f' have a (finite) bound aperture at 1? Write your answer in interval notation.

Homework Equations

X

The Attempt at a Solution

Honestly, I accept to clue where to start; especially in regards to the removable discontinuity. I tried making the functions equal each other for the jump discontinuity but, that was way out every bit far as I can tell.

--------------------------

If I could at least be given a starting point to go from, it would be appreciated. I'm non asking for the answers only, I'yard so lost and just staring at this thing isn't helping. Thanks,

RK

Answers and Replies

If a function has a removable discontinuity at [tex] ten = a [/tex], that means these things.

[tex]
\begin{marshal*}
\lim_{x \to a^+} f(x) & = \lim_{x \to a^-} f(x)\\
\text{ so } & \lim_{x \to a} f(x) \text{ exists}\\
f(a) & \text{ is divers} \\
f(a) & \ne \lim_{x \to a^+} f(x)
\stop{align*}
[/tex]

All the limits in a higher place are finite. Basically, if there is a removable discontinuity at [tex] x = a [/tex], the part has almost everything needed to be continuous in that location, simply the function value is "incorrect". Here is an example.

[tex]
f(ten) = \begin{cases}
3x+five & \text{ if } 10 \ne 10\\
twenty & \text{ if } x = x
\end{cases}
[/tex]

Hither the limit at 10 is equal to 35 (because both one-sided limits equal 35), but [tex] f(ten) = xx [/tex], so the function is not continuous in that location. We say in that location is a removable discontinuity at 10 considering we tin can practice this: brand [tex] f [/tex] continuous just past doing this:

[tex]
F(x) = \brainstorm{cases}
3x + 5 & \text{ if } x \ne 10\\
35 & \text{ if } x = ten
\stop{cases}
[/tex]

i.e. - we remove the discontinuity by redefining the role at the problem point, in guild to make the function continuous there.

A function has a jump aperture at a spot if the two one-sided limits both exist but are not equal. Here is an example - the jump discontinuity is at [tex] ten = 2 [/tex].

[tex]
west(x) = \begin{cases}
2x - i & \text{ if } x <=ii\\
10x + ane & \text{ if } ten > two
\end{cases}
[/tex]

The limit from the left is three, the limit from the correct is 21. If y'all were to view the graph of [tex] west [/tex] you lot would see a 'jump' in the two portions of the graph at [tex] x = 2 [/tex]. What yous need to do is find the values of [tex] a [/tex] and [tex] b [/tex] to make your function take the types of beliefs discussed here. Good luck.

If you could please relate that back to my question, information technology would be appreciated. Over again, no answers only, I'yard still a little confused and don't know where to start.
If a part has a removable aperture at [tex] x = a [/tex], that means these things.

[tex]
\brainstorm{align*}
\lim_{10 \to a^+} f(ten) & = \lim_{ten \to a^-} f(x)\\
\text{ so } & \lim_{x \to a} f(x) \text{ exists}\\
f(a) & \text{ is defined} \\
f(a) & \ne \lim_{x \to a^+} f(x)
\stop{align*}
[/tex]

luck.

Information technology is also the case where f(a) is non defined but the overall limit equally x-->a exists. The way i can remove this is by defining the role at x=a.

P.S. I am sure you know this, simply this is adressed to the OP.

Related Threads on How do I determine where a office has a removable and a jump discontinuity?

  • Last Post
  • Last Post
  • Last Post
Mark44
  • Terminal Post
  • Concluding Post
STEMucator
  • Concluding Post
  • Last Post
  • Last Mail service
  • Last Post
  • Last Post
micromass
  • Forums
  • Homework Assistance
  • Calculus and Beyond Homework Help

Source: https://www.physicsforums.com/threads/how-do-i-determine-where-a-function-has-a-removable-and-a-jump-discontinuity.262486/

Posted by: robertsgois1936.blogspot.com

0 Response to "How To Find A Removable Discontinuity"

Post a Comment

Iklan Atas Artikel

Iklan Tengah Artikel 1

Iklan Tengah Artikel 2

Iklan Bawah Artikel